LSAT and Law School Admissions Forum

Get expert LSAT preparation and law school admissions advice from PowerScore Test Preparation.

 jbeyer
  • Posts: 6
  • Joined: Jul 18, 2011
|
#1876
Hi guys,

I need some help with this:

Political theorist: The chief foundations of all governments are the legal system and the police force; and as there cannot be a good legal system where the police are not well paid, it follows that where the police are well paid there will be a good legal system.

The reasoning in the argument is not sound because it fails to establish that...

I understand why D is the correct answer (a well-paid police fore is sufficient to guarantee a good legal system).

What I am having trouble with is necessary and sufficient. For the life of me I cannot figure out what the necessary and sufficient parts are.

Part 2 of sentence 1 and as there cannot be a good legal system (not sufficient) where the police are not well paid (not necessary). Then the contrapositive is if there is a well paid police (sufficient) then there is a good legal system (necessary).

THen there is another sufficient necessary clause in the last part of the sentence. it follows that where the police are well paid (sufficient) there will be a good legal system(necessary)

This is my reasoning although I know it is wrong. I just can't seem to figure it out. I can put if and then either way and both hypotheticals make sense. What is the way to figure the necessary and sufficient out when there is no obvious choice (i.e. they don't use words like if or only or only if...)

Thanks so much,

Jonathan
 Adam Tyson
PowerScore Staff
  • PowerScore Staff
  • Posts: 5153
  • Joined: Apr 14, 2011
|
#1877
Jonathan,

The key here is the indicator word "where" in that second sentence, which indicates a sufficient condition, rather than a necessary condition (just like "when" or "in the case that" would do). "Where" introduces "not well paid police" (so "if Not WPP"), and the rest is the necessary condition ("then Not GLS"). Diagram is NOT WPP ---> NOT GLS. The contrapositive is GLS---> WPP. The author made a mistaken negation of the original relationship.

Hope that helps!

Adam
 jbeyer
  • Posts: 6
  • Joined: Jul 18, 2011
|
#1886
Thank you so much!!

So in any situation that would involve necessary and sufficient, the word where would indicate a sufficient?

IS there a more comprehensive list of these words? I can understand that phrases like "in the case that" is more rare then words like when or where.

Jonathan
User avatar
 Dave Killoran
PowerScore Staff
  • PowerScore Staff
  • Posts: 5853
  • Joined: Mar 25, 2011
|
#1887
Hey Jonathan,

Yes, "where" often will introduce a sufficient condition, although you have to be careful for situations such as "only where," which would be a necessary condition. Of course, as you note, you have to be in a conditional environment for this to work. Sentences such as "The geologist was not sure where the fault line had shifted" or "Where are the the people who objected to the plan now?" aren't really going to be conditional. On the other hand, "The geologist deduced that where the fault line shifted there was damage to the crust" is going to be conditional.

That last point brings up why there isn't a "complete" list of conditional indicators. First, the English language is so flexible that there are a number of cases where a word can be conditional in one instance but not in another. Memorizing a list would lead some people into deep trouble because they'd see a particular word from the list and then automatically go into diagramming mode. Second, such a list would be so lengthy as to be more or less useless. We start with the basic list of indicators because it covers about 95% of the cases you see on the LSAT, and by starting with that list you can get a feel for how they introduce these ideas and manipulate them. Over time, what should happen is that you begin to understand the underlying idea of how conditionality works so you can recognize it no matter what words or phrases they use to introduce it.

If it helps, one way to think about sufficient conditions is that they apply to a group or a set of cases. Words like "all" and "every" specifically number the group, words like "if" indicate that when the cases happen there is a conditional that accompanies that occurrence, and words like "where" indicate a range of occurrences (much like "all" or "every" without the absolute element). Thus, when you see language that indicates that a group of elements or a set of cases is being addresses, you often will have a conditionality (or causality, but the language used with be different, and more active).

Please let me know if that helps. Thanks!
 JennuineInc
  • Posts: 18
  • Joined: May 11, 2016
|
#24635
Hi

I have a question about the Question Stem. When it says "The reasoning in the argument is not sound because it fails to establish that" I get confused by the wording. It is asking for what the reasoning itself DOES NOT establish or is it asking for what the author makes a mistake in establishing?

Because the answer seems like its saying the author makes the mistake (fails?) in establishing that "a well-paid police force is sufficient to guarantee a good legal system"

But the question to me seems like it's saying "the reasoning in the argument is not sound because it is missing (Fails) to establish that"

I might be reading into it too much but it's confused me once before whenever the Question Stem asks what the argument FAILS TO ESTABLISH.

Thanks!
 David Boyle
PowerScore Staff
  • PowerScore Staff
  • Posts: 836
  • Joined: Jun 07, 2013
|
#24944
JennuineInc wrote:Hi

I have a question about the Question Stem. When it says "The reasoning in the argument is not sound because it fails to establish that" I get confused by the wording. It is asking for what the reasoning itself DOES NOT establish or is it asking for what the author makes a mistake in establishing?

Because the answer seems like its saying the author makes the mistake (fails?) in establishing that "a well-paid police force is sufficient to guarantee a good legal system"

But the question to me seems like it's saying "the reasoning in the argument is not sound because it is missing (Fails) to establish that"

I might be reading into it too much but it's confused me once before whenever the Question Stem asks what the argument FAILS TO ESTABLISH.

Thanks!

Hello JennuineInc,

It could be either, in any given case. The author may not bother to establish something; or he/she may try to establish it, but in an invalid way.

Hope this helps,
David
 LSAT2018
  • Posts: 242
  • Joined: Jan 10, 2018
|
#45432
I would like to ask confirmation on this:
for this Flaw in the Reasoning question, the answer would be diagrammed as Well Paid Police Force → Good Legal System
User avatar
 Dave Killoran
PowerScore Staff
  • PowerScore Staff
  • Posts: 5853
  • Joined: Mar 25, 2011
|
#45490
LSAT2018 wrote:I would like to ask confirmation on this:
for this Flaw in the Reasoning question, the answer would be diagrammed as Well Paid Police Force → Good Legal System
Yes, you are correct! "A well-paid police force is sufficient to guarantee a good legal system" diagrams as Well Paid Police Force → Good Legal System.

Good job!
 soobin903
  • Posts: 4
  • Joined: Nov 13, 2019
|
#71929
Hi,
I have a question on how D is to be diagrammed.

I have been able to deduce 'Good Legal System -> Well Paid Police' from the passage.
Since the question is asking how the political theorist's Mistaken Reversal fails to establish 'GLS->WPP', the answer should show that GLS is the sufficient condition and that WPP is the necessary condition.

However, in D, WPP is referred to as the sufficient condition... thus establishing yet another mistaken reversal.

Am I to understand the clause preceding 'sufficient to guarantee' as the necessary condition and not as the sufficient condition, or am I missing out on something here?

Thanks in advance!
User avatar
 KelseyWoods
PowerScore Staff
  • PowerScore Staff
  • Posts: 1079
  • Joined: Jun 26, 2013
|
#71936
Hi soobin903!

You are correct that the premise should be diagrammed as:

Good Legal System :arrow: Police Well Paid

The conclusion, then, is diagrammed as:

Police Well Paid :arrow: Good Legal System

The author made a Mistaken Reversal in determining that conclusion. Just because a well-paid police force is necessary for a good legal system (as our premise tells us it is) does not mean that it is sufficient for a good legal system. So the flaw is that the author has failed to establish that a well-paid police force is enough to guarantee a good legal system. That's what we're looking for in the correct answer and that's what answer choice (D) gives us (Police Well Paid :arrow: Good Legal System).

Your prephrase here was incorrect. We are not looking for an answer choice that gives us the conditional relationship GLS :arrow: WPP. That is the relationship in the premise and that was established by the author. It's the conclusion (WPP :arrow: GLS) that was not established by the author and so that's what we are looking for in the correct answer.

Hope this helps!

Best,
Kelsey

Get the most out of your LSAT Prep Plus subscription.

Analyze and track your performance with our Testing and Analytics Package.